Washington Supreme Court Holds No Constitutional Mandate to Determine Competency to Represent Self

In re Rhome, 2011 Wash. LEXIS 743 (September 15, 2011)

The Washington Supreme Court has held that a trial court is not constitutionally required to independently determine whether a defendant was sufficiently competent to waive counsel when he had previously been found competent to stand trial following a pre-trial hearing. The court held that a defendant’s mental health status is but one factor a trial court may consider in determining whether a defendant has knowingly and intelligently waived his right to counsel and to represent himself.

In this case, Rhome was charged with first degree murder with a deadly weapon of a 17- year old girl. Another juvenile confessed to stabbing the girl but identified the defendant as the “mastermind” behind the killing. Since early childhood, the defendant had been treated for psychiatric disturbances, including several in-patient psychiatric hospitalizations. He had received a myriad of diagnoses including, psychotic disorder, delusional disorder, oppositional defiant disorder, mild mental retardation, obsessive/compulsive traits, and pervasive developmental disorder (Aspergers disorder). The trial court held a competency hearing finding that the defendant had not proved he was incompetent to stand trial. Throughout the pre-trial proceedings, the defendant asserted his right to represent himself. The court first denied his request to proceed pro se indicating that his ability to do so was equivocal. After his renewed request, the court advised him of the risks and engaged in coloquy to determine if he understood the significance of this undertaking. His mental health issues were not specifically addressed during the colloquy. The court granted his request and appointed standby counsel to assist him. The jury convicted the defendant and he was sentence to 30 years in prison. A mental health expert for the defense who later examined the defendant’s performance in representing himself testified that his mental illness impacted his ability to defend himself in court. He testified that the defendant engaged in perseverative and aggressive questioning that was incoherent or intimidating, and he was unable to self-regulate his emotions and behavior.

In June 2008, just following the state courts’ denial of the defendant’s direct appeals, the United States Supreme Court decided Indiana v. Edwards, 554 U.S. 164 (2008). In Edwards, the Supreme Court held that a trial court could insist that a defendant proceed with counsel even though the court had found the defendant was competent to stand trial. The Washington Court stated that the Edwards decision assumes that a defendant will “assist” in his defense, not “conduct” his defense when the defendant has been found competent to stand trial. Competency to stand trial does not equate with the right to represent oneself and the Supreme Court declined to set a standard for the state to follow. In determining whether a defendant has the right to waive counsel, the court considers his background, experience and conduct, which may include his history of mental illness. In denying his petition for post-conviction relief, the Washington Supreme Court held that a defendant’s mental health status is but one factor a trial court must consider in determining whether a defendant has knowingly and intelligently waived his right to counsel. An independent determination of competency for self-representation is not a constitutional mandate.

Found in DMHL Volume 31 Issue 1

Maine Finds Right to Competency in Post-Conviction Proceedings

Haraden v. State, 32 A.3d 448 (Maine 2011)

The Maine Supreme Judicial Court ruled on November 17, 2011 that a convicted defendant has the statutory right to be competent during post-conviction proceedings. Although a defendant has no constitutional right of access to post-conviction proceedings to overturn his or her conviction, Maine has statutorily created a process whereby inmates may challenge their convictions, including setting time limits within which relief may be sought, the number of petitions that may be filed, the nature and scope of claims that may be pursued, and the type of relief that may be granted.15 M.R.S § 2130 (2010). As part of the post-conviction process, inmates are specifically given a statutory right to counsel. The Court therefore found that implied within that right to counsel is the right to the effective assistance of counsel. It then reasoned that counsel cannot effectively assist his client if his client cannot meaningfully communicate with him.

In this case, the inmate was convicted of murder by a jury and sentenced to 52 years in prison. After his trial, conviction and appeal, the inmate raised factual allegations that he was denied the effective assistance of counsel. On the inmate’s motion, the court ordered a mental evaluation by the State Forensic Service. The evaluation indicated that although the inmate was not psychotic, he was unable to assist his attorney in the post-conviction process. Based upon the evaluation, the trial court found him incompetent to proceed. The court then proceeded to decide the matter upon the legal issues presented, but continued those claims based upon factual contentions until such time as the inmate became competent. During that time period, the inmate was ordered to remain in the Department of Corrections and not be transferred to the Department of Health and Human Services for restoration to competency.

In upholding the right found by the trial court, the Supreme Court was faced with a dilemma of how to proceed when an inmate may have a legitimate claim for release but cannot pursue it due to his incompetency. It therefore had to fashion a process to handle the prisoner’s claims. The Court determined that when an inmate’s competency is in question, the court must order an evaluation by the State Forensic Service. Because a defendant was presumed to be competent during trial, the burden rests on the inmate to prove his incompetency by a preponderance of the evidence. If he does so, the court must still proceed to adjudicate the inmate’s claims and defense counsel must represent the inmate to the best of his or her ability. Under Maine post-conviction law, an inmate must 1) file a post-conviction claim within one year and 2) may only seek post-conviction review once, raising all claims he may have in that petition or else they are considered waived. If the inmate is found to be incompetent, however, the Court then provided that an inmate may file an affidavit at a later date alleging that he had previously been found incompetent and has regained his competence as a result of the passage of time, medical intervention or some other substantial change. If the court then determines the inmate has regained competency, it must review the petition to determine whether, if the newly asserted evidence or grounds were true, the outcome of a post-conviction judgment would be different, and which, if any, of the defendant’s claims may be pursued despite the intervening delay. The Supreme Court agreed with the trial court that the inmate must remain in the custody of the Department of Corrections during the period of incompetency and not be transferred to the Department of Health and Human Services for restoration services. Presumably, the inmate could not then be ordered treated over his objection to restore him to competency.

Found in DMHL Volume 31 Issue 2

California Supreme Court Rules Court Has Discretion Whether to Permit Competent Defendant to Represent Self

People v. Johnson, 2012 Cal. LEXIS 600 (January 30, 2012)

Following the United States Supreme Court decision in Indiana v. Edwards, 554 U.S. 64 (2008), the California Supreme Court has ruled that trial courts may deny the right to represent themselves to defendants who fall into a “gray area” between those who are competent to stand trial and those who are competent to conduct their own trial. In so ruling, the Court found that California law has never afforded defendants the right to represent themselves, but only permits self-representation in noncapital cases in the judge’s discretion. In capital cases, the law specifically requires defendants to be represented by counsel at all stages of the proceedings. The legislative history states that pro se litigants cause unnecessary delays at trial and generally disrupt the proceedings. The history further provides that the burden on the justice system is not outweighed by any benefits to defendants who generally gain nothing by representing themselves.

Prior to Edwards, and because California courts are required to follow federal constitutional law, they afforded criminal defendants the right to represent themselves in spite of the state law based upon the United States Supreme Court decision in Faretta v. California, 422 U.S. 806 (1975), holding that defendants have a Sixth Amendment Constitutional right to selfrepresentation. California courts further presumed that a defendant’s right to self-representation was absolute based upon Godinez v. Moran, 509 U.S. 389 (1993). That case held that a defendant found competent to stand trial was allowed to waive counsel and plead guilty, rejecting the argument that federal law required a higher standard of competence for waiving counsel or pleading guilty than to stand trial. In 2008, the United States Supreme Court held in Edwards that a trial court could insist that a defendant proceed with counsel even though the court had found him competent to stand trial.

In this case, Johnson was charged with two separate assaults, one an early-morning brutal sexual assault on a bar tender and later the same day, hitting the patron of a sandwich shop with a metal chair rendering him unconscious. A single judge presided over all the proceedings. The defendant was originally represented by counsel but early on requested to represent himself, which request was granted. During pre-trial proceedings, the defendant conducted himself in an unusual manner and the nature and content of letters he filed with the court and others cast doubt on his competence. The judge thereupon appointed counsel to represent the defendant in competency proceedings and ultimately appointed three mental health experts to evaluate him. The defendant refused to be interviewed by any of the experts who then testified that they were unable to state with certainty whether he was competent or not. His behavior in court and jail, and his bizarre filings led one expert to proffer a diagnosis of delusional disorder with conspiracy paranoia and to strongly suspect incompetency. Based upon all of the evidence, the jury found the defendant competent to stand trial. The trial judge, however, found the defendant incompetent to represent himself and appointed counsel for him.

The California Supreme Court held that competency to stand trial was a matter to be decided by a jury, but competency to represent oneself was a decision within the sound discretion of the trial judge. The Court stated that although courts should be cautious about making competency decisions without the benefit of expert evidence, the judge’s own observations of the defendant’s behavior supported a common sense finding of incompetence. Here the court was able to observe the defendant’s behavior in representing himself over several months; the defendant had already refused mental health evaluations on competency to stand trial; and the judge placed on the record examples of his disorganized thinking, deficits in sustaining attention and concentration, impaired expressive abilities, anxiety and other common symptoms of severe mental illness. The Court found the trial court had the discretion to determine the defendant lacked the competency to represent himself and the judge had not abused his discretion in this case.

Although urged to do so by the parties and amicus curiae, the California Supreme Court, like the United States Supreme Court in Edwards, declined to set out a standard for determining competency to represent oneself, but decided that trial courts should simply determine whether the defendant suffers from a severe mental illness to the point where he or she cannot carry out the basic tasks needed to present the defense without the help of counsel. The decision in this case is similar to the Washington Supreme Court decision in In Re Rhome, 260 P.3d 874 (Wash. 2011) and reported in the last issue of Developments in Mental Health Law.

Found in DMHL Volume 31 Issue 2

Ninth Circuit Holds District Court Decision Refusing to Seal Competency Proceedings Not Subject to Interlocutory Appeal

United States v. Guerrero, 2012 U.S. App. LEXIS 18504 (9 th Cir. Cal. Aug. 31, 2012)

The Ninth Circuit Court of Appeals has refused to hear the interlocutory appeal of the district court’s denial of defendant’s motion to seal his competency proceedings under the collateral order doctrine.

James Guerrero and a co-defendant were indicted in 2008 for the first degree murder of a United States correctional officer and the government filed a notice of intent to seek the death penalty. In April 2011, the defendant filed a motion for a hearing to determine his competency to stand trial, along with six exhibits, including a 27-page social history and a 77-page memorandum describing defense counsels’ interactions with the defendant. The defendant moved the court to seal the evidentiary hearing, all exhibits, post-hearing briefs and any detailed findings of fact. The district court denied the motion to seal the proceedings and documents, finding an overriding public interest in criminal competency proceedings, and scheduled the competency hearing. The court did issue a protective order prohibiting certain privileged and confidential information from being used at trial. The defendant thereupon sought an interlocutory appeal to the Ninth Circuit.

As a general rule, an appellate court may hear appeals only from a district court’s final decision concluding litigation. A court may review an intermediate decision under the collateral order doctrine only when the decision 1) has conclusively determined the disputed question, 2) resolved an important issue completely separate from the merits of the case, and 3) would be effectively unreviewable on appeal from a final judgment. A decision that is effectively unreviewable on appeal is one that would imperil a substantial public interest or some particular value of high order, public access to criminal competency proceedings being one such interest.

The defendant argued in this case that once his personal history was publicly disclosed at the competency hearing it would intrude on the attorney/client privilege, taint the jury pool and invade his and his family’s privacy, and was thus effectively unreviewable. The Ninth Circuit found in this case that the first two prongs of the standard had been met. The issue of whether to seal the competency proceedings had been conclusively determined and the competency issue was separate on the merits from the criminal proceeding. However, the Court found that the decision whether to seal the competency proceeding could be adequately vindicated on appeal through reversal of a conviction and the ordering of a new trial. Moreover, other avenues were also available to protect the defendant’s interests including a petition for writ of mandamus asking a court to seal the records, rigorous jury screening, possible relocation of the trial, and entry of protective orders, prohibiting the use or disclosure of certain documents. The Ninth Circuit therefore denied the defendant’s appeal and remanded the case for a determination of the defendant’s competency and further trial on the merits.

Found in DMHL Volume 31 Issue 6

Defendant Detained in Mental Health Facility for Restoration to Competency is Prisoner under Prison Litigation Reform Act

Gibson v. City Municipality of New York, 692 F.3d 198 (2012)

The Second Circuit Court of Appeals has upheld the district court’s dismissal of a petitioner’s motion to proceed in forma pauperis, resulting in dismissal of his complaint against a number of city, corrections and mental health officials alleging they violated his civil rights. The petitioner had filed three previous petitions as a prisoner that had been dismissed as frivolous, malicious, or failed to state a claim upon which relief may be granted. The Court held that although the petitioner was being detained in a mental health facility, he was still a “prisoner” for purposes of the Prison Litigation Reform Act (“PLRA”), 28 U.S.C. § 1915(g), and thus subject to the Act’s limitation on proceeding in forma pauperis in federal court.

The petitioner, Bennie Gibson, had been charged with third degree criminal mischief under New York law and was being detained at Kirby Psychiatric Center in the custody of the Commissioner of Mental Health on a temporary order of observation for restoration of his capacity to stand trial. While federal law generally permits a district court to waive filing fees for individuals who cannot pay and to proceed in forma pauperis, Congress enacted the Prison Reform Litigation Act in 1995 to limit abuse of the legal system by prisoners who file repetitive frivolous complaints. “Prisoner” is defined under the Act as “any person…detained in any facility who is accused of…violations of criminal law.” 28 U.S.C. § 1915(h). Under New York law, criminal charges are not dismissed against a defendant held in the temporary custody of the Commissioner of Mental Health, but are merely suspended pending his treatment and restoration to capacity. Gibson therefore met the definition of a prisoner as a person detained as a result of an accusation, conviction, or sentence for a criminal offense. Had Gibson been held under a final order of observation as a civil unrestorable patient, been found not guilty by reason of insanity, or been civilly committed as a sexually violent predator, the result may not have been the same.

Found in DMHL Volume 32 Issue 1

US Supreme Court Holds Incompetence of State Prisoner Does Not Suspend Federal Habeas Proceeding

Ryan v. Gonzales, 568 U.S. __(2013)
slip opinion available at: http://www.supremecourt.gov/opinions/12pdf/10-930_7k47.pdf . Justice Clarence Thomas wrote the decision for a unanimous court.

The United States Supreme Court decided on January 8, 2013 in two consolidated capital cases that the incompetence of state prisoners does not suspend federal habeas corpus proceedings under either 18 U.S.C. § 3599 or 18 U.S.C. § 4241, reversing the decisions of the Ninth and Sixth Circuit Courts of Appeal.

An Arizona jury convicted Ernest Valencia Gonzales of felony murder, armed robbery, aggravated assault, first-degree burglary and theft. Gonzales had repeatedly stabbed a husband and his wife in front of their 7-year-old son during a burglary of their home. The husband died but his wife survived after several days of intensive care. The trial court sentenced Gonzales to death on the murder charge and to prison terms on the other charges.

After exhausting his state court remedies, Gonzales filed a petition for writ of habeas corpus in federal district court. Gonzales’ appointed counsel filed a motion to stay the petition on the grounds that Gonzales was no longer capable of communicating or assisting his counsel. The Ninth Circuit had previously held in Rohan v. Woodford, 334 F.3d 803 (9th Cir. 2003), that 18 U.S.C. § 3599(a)(2), the federal statute guaranteeing state capital prisoner’s a right to counsel in federal habeas proceedings required that the petitioner be sufficiently competent when he raises claims that could potentially benefit from his ability to communicate with counsel. If he is not competent, he is entitled to a stay of the proceedings pending his restoration to competency. The Ninth Circuit reasoned that without a stay, the petitioner is denied his right to counsel. Although applying Rohan, the district court, nevertheless, denied Gonzales a stay. It determined that the claims he raised were based on the record before the trial court or were resolvable as a matter of law ,and his lack of competence would therefore not affect his counsel’s ability to represent him. Gonzales then filed an emergency petition for writ of mandamus in the Ninth Circuit. While his case was pending, the Ninth Circuit decided Nash v. Ryan, 581 F.3d 1048 (2009), holding that habeas petitioners have an absolute right to competence on appeal, even though appeals are entirely record-based. The Ninth Circuit thereupon granted Gonzales’ writ and entered a stay pending his competency determination. The Supreme Court granted Arizona a writ of certiorari.

In the second case, an Ohio jury convicted Sean Carter of aggravated murder, aggravated robbery, and rape, and sentenced him to death for anally raping his adoptive grandmother and stabbing her to death. After exhausting his state court appeals, Carter’s attorney filed a federal habeas petition along with a motion requesting a competency determination and a stay of his proceedings. Following several psychiatric evaluations, the district court found Carter incompetent to assist counsel and, applying the Ninth Circuit’s test in Rohan, finding that Carter’s assistance was necessary to develop four of his exhausted state court claims. As a result, the district court then dismissed the habeas petition without prejudice and tolled the stature of limitations under the Antiterrorism and Effective Death Penalty Act of 1996.

On appeal, the Sixth Circuit recognized that federal habeas petitioners do not have a constitutional right to competence, but found a statutory right to competence under 18 U.S.C. §4241, relying in part on the Supreme Court’s decision in Rees v. Peyton, 384 U.S. 312 (1966). Rees required that a Virginia habeas petitioner awaiting the death penalty who decided to forego any further appeals of his conviction or sentence be competent enough to understand the nature of the proceeding and assist counsel before he could withdraw his habeas petition. The Sixth Circuit then ordered that Carter’s petition be stayed indefinitely with respect to any claims that required his assistance. The Supreme Court granted Ohio’s petition for writ of certiorari and consolidated the two cases for review.

On review, the Supreme Court noted not only that there is no constitutional right to counsel in habeas proceedings, but there is no due process right at all to collateral review. Murray v. Giarratano, 492 U.S. 1, 10 (1989). It acknowledged that the statute, 18 U.S.C. § 3599(a)(2), grants federal habeas petitioners on death row the right to federally funded counsel. It also gives district courts the power to authorize investigative, expert and other services. But the Court found the statute does not require district courts to stay proceedings when habeas petitioners are found incompetent. The Court reasoned that the right of a criminal defendant to competence in the original trial flows from the Due Process Clause of the Fourteenth Amendment and not from the Sixth Amendment right to counsel, even though the right to counsel at trial may be compromised if the defendant is not able to communicate with counsel. Review of a state court conviction in a federal habeas proceedings is limited to the record in existence at the time of the state court trial. Given the backward-looking, record-based nature of habeas proceedings, counsel can therefore effectively represent the petitioner regardless of his competence. The Court went on to find that the Ninth Circuit decision in Rohan incorrectly relied on Rees, a decision which simply dealt with an incompetent capital petitioner’s ability to withdraw his petition for certiorari.

Also reviewing the Sixth Circuit’s conclusion that 18 U.S.C. § 4241 provides a statutory right to competence, the Court found that § 4241 does not even apply to habeas proceedings involving state prisoners. Section 4241 only applies to federal defendants and probationers subject to prosecution by the United States and only to trial proceedings prior to sentencing, or after probation or supervised release. The Court therefore held that neither 18 U.S.C. § 3599 nor § 4241 requires suspension of a capital petitioner’s federal habeas proceeding when he has been adjudicated incompetent.

Both Gonzales and Carter also argued that district courts have the equitable power to stay proceedings when they determine habeas petitioners are incompetent. In Gonzales’ case, the Supreme Court held that the district court correctly found that all of his claims were record-based or resolvable as a matter of law. The district court did not therefore abuse its discretion in denying the stay. In Carter’s case, the district court found that four of his claims could potentially benefit from his assistance. However, the Supreme Court determined that three of the claims were adjudicated in state court post-conviction proceedings and could be reviewed on the record. It found it unclear whether the fourth claim, alleging ineffective assistance of appellate counsel for failing to raise trial counsel’s failure to pursue a competency to stand trial issue, required consultation with counsel. The Court nevertheless held that an indefinite stay would be inappropriate under the Antiterrorism and Effective Death Penalty Act whose purpose is to reduce delay in the execution of state and federal criminal sentences. The Court remanded Carter’s case with instructions that if the court found the fourth claim would substantially benefit from his assistance, the court must take into account the likelihood that Carter will regain competence in the foreseeable future. If there is no reasonable hope of competence, a stay is inappropriate. In a footnote, the Court acknowledged that its opinion does not implicate the prohibition against execution of a death sentence for a prisoner who is insane.

Found in DMHL Volume 32 Issue 1

Sixth Circuit Finds Special Circumstances Preclude Involuntary Medication of Incompetent Defendant Charged with Bank Robbery

United Sates v. Grigsby, 712 F.3d 964 (6th Cir. 2013)

The Sixth Circuit Court of Appeals held on April 11, 2013 that special circumstances exist that outweigh the government’s interest in prosecuting for bank robbery a pre-trial detainee to restore him to competency. Unlike the Eighth Circuit evaluating similar special circumstances in United States v. Mackey, 717 F.3d 569 (8th Cir. 2013), reviewed below, a majority three-judge panel concluded that the potential availability of lengthy civil commitment together with the likelihood that, even if the defendant is restored to competency, he will be found not guilty by reason of insanity, greatly diminishes the government’s interest in prosecution.

Dennis Grigsby was charged with three counts of unarmed bank robbery in Columbus, Ohio, between January and March 2010. Grigsby’s attorney requested the court to order mental evaluations to determine Grigsby’s competence to stand trial and his sanity at the time of the offenses. The district court granted the motion and he was transferred to the Metropolitan Correctional Center in New York where two psychologists conducted the examinations. They both diagnosed Grigsby with paranoid schizophrenia and determined him incompetent to stand trial, but one postulated that he was sane at the time of his offense and the other that he was not. Both psychologists reported that Grigsby’s mental disease did not significantly interfere with his appreciation of the wrongfulness of his acts, but there was insufficient information about whether mental illness impaired his ability to appreciate the wrongfulness of his conduct. Neither the government nor the defendant objected to the reports’ findings and the district court committed Grigsby to the custody of the Attorney General in November 2010 for a period not to exceed four months for a determination whether he could be restored to competency.

Grigsby was then transferred to the Federal Medical Center in Butner, North Carolina and was evaluated by a psychiatrist and psychologist at the facility. They found that Grigsby had a normal upbringing, education and employment until he stopped working due to “job burnout.” He was convicted of grand theft auto, disorderly conduct, and resisting arrest in 2006; for criminal trespassing in 2007; and for resisting arrest in 2010. He served short jail sentences for these crimes. He was also charged with voyeurism and menacing by stalking, which were not prosecuted. Grigsby was in good physical health, never received mental health treatment and was not taking antipsychotic medication for his illness. He followed all of the rules of the facility, got along well with peers and staff, was not gravely disabled and did not present a danger to self or others, or to the safe operation of the facility. Although his dress and grooming were appropriate and he was oriented to person, place, time and circumstances, and denied hallucinations and delusions, they reported, however, that Gillenwater’s conversation was not linear and he displayed substantial evidence of thought disorder, including an extensive, but poorly organized, paranoid religious delusional system extending into all major functional areas of his life.

The evaluators determined that Grigsby was incompetent to stand trial. Because he was refusing all antipsychotic medications, they also requested an order under Sell v. United States, 539 U.S. 166 (2003), allowing them to medicate him involuntarily to restore his competence to stand trial. The evaluators both determined that antipsychotic medication was substantially likely to render Grigsby competent to stand trial and substantially unlikely to produce sideeffects that would interfere with his ability to assist his attorney in conducting a defense and that less intrusive therapies, such as psychotherapy would not able work. They reported that antipsychotic medication was medically appropriate and would take at least four months to be effective.

In determining whether to uphold the district court’s order authorizing involuntary medication to restore Grigsby to competence, the Sixth Circuit applied the Sell test requiring the government to prove by clear and convincing evidence that 1) an important government interest in prosecution exists; 2) involuntary medication will significantly further the governmental interest, which requires proof both that administration of the medication is substantially likely to render the defendant competent to stand trial and substantially unlikely to cause side effects that will interfere significantly with the defendant’s ability to assist counsel in conducting the trial defense; 3) involuntary medication is necessary to further the governmental interest; and 4) administration of drugs is medically appropriate for the defendant. Id at 180-81.

At the district court hearing, Grigsby conceded that the government had an important interest in bringing him to trial for the serious crime of bank robbery. Grigsby argued, however, that special circumstances existed in his case to diminish that interest. He first argued that the potential availability of lengthy civil commitment together with the likelihood that he would be found not guilty by reason of insanity addressed the government’s interest in his continued confinement.

The Sixth Circuit reviewed the Butner psychiatrist’s testimony that if Grigsby is not forcibly medicated he would remain psychotic and medical staff at FMC-Butner would request his civil commitment. In order to be civilly committed, federal law requires the district court to determine by clear and convincing evidence whether Grigsby is suffering from a mental disease or defect and poses a substantial risk of bodily injury or serious damage to property. The evidence revealed that although Grigsby was not a present danger to himself or others in the structured environment at Butner, the government psychiatrist testified that he was not necessarily fit for release into society. The district court found that the evidence was inconclusive on that issue, but the Sixth Circuit determined that the district court should have inquired further. The Sixth Circuit found evidence in the record that supported the possibility that Grigsby might meet the insanity standard at trial, if restored to competence. Both the government’s psychiatrist and Grigsby’s expert agreed that Grigsby would need to be restored to competence before a definitive determination could be made, but Grigsby’s expert testified that he suffered from a severe and chronic mental illness and likely suffered from it at the time of the bank robberies. He also surmised that Grigsby may have experienced previous psychotic episodes.

The Court further determined that the length of Grigsby’s confinement while the government attempts to restore him to competency and prosecute him may approximate the length of any prison sentence he might receive if convicted. If convicted, the government indicates Grigsby might receive a sentence of 57 to 71 months based on sentencing guidelines. Unlike the Eighth Circuit, the Sixth Circuit found the government’s analysis under the guidelines instructive because the government often uses this range, rather than the maximum possible sentence, as a basis for negotiating plea agreements. The Court therefore found this range more useful when, as here, the government advances the length of sentence as a core reason why it wants to prosecute. The Court found that Grigsby had already been held since July 2010, or 33 months. It would take at least four months to restore him to competency, plus additional time to bring him to trial, and potentially additional time to re-restore him if he loses competency during the pendency of the trial. The Court also noted that often defendants plead guilty after they are restored to competency which reduces further their period of imprisonment under the guidelines. All of these factors indicated to the Court that Grigsby may remain in custody for a period roughly equivalent to the length of any prison sentence he might serve, thus lessening the government’s interest in prosecuting him.

The Court went on to find that antipsychotic medication can burden a defendant’s fair trial rights by affecting his ability to comprehend and react to trial events. Grigsby argued that he had trial-related concerns that tardive dyskinesia and akathisia, which causes constant movement and an inability to remain still, might impair his ability to make a dignified appearance before a jury and assist his counsel in his defense. Although the Court noted that the record indicates antipsychotic medication is generally effective in restoring competency especially in patient’s with Grigsby’s positive symptoms, it also found that the government’s psychiatrist testified that 30% of patients do not respond to haloperidol and another 30% show only a partial response. He also testified that 30% of individuals treated with haloperidol develop pseudoparkinsonism, 20-30% develop akathisia, 2-10% develop acute systonic reactions, and 18- 40% develop irreversible tardive dyskinesia. Although Grigsby had never previously been treated with antipsychotic medication, and the government psychiatrist testified that other medications would be prescribed to counter the side-effects and that the medication would be changed or discontinued if the side effects continued or irreversible side-effects developed, the Court nonetheless found that the record lacked clear and convincing evidence that medication is substantially unlikely to cause side effects that will interfere with Grigsby’s ability to assist in his defense.

Based upon all the facts above, the majority of the Sixth Circuit three-judge panel hearing the appeal found that the findings of the district court supporting an order authorizing involuntary medication were clearly erroneous. The Court reversed the district court order and remanded the case for further proceedings, specifying its expectation that the district court would determine whether civil commitment is appropriate for Grigsby. A dissent was filed in this case stating that the Court majority’s analysis of the special circumstances was highly speculative as to the likelihood of Grigsby’s civil commitment, his being found not guilty by reason of insanity, his pretrial confinement exceeding any sentence he might receive, and any side-effects impairing his pre-trial rights. Compare this decision with the decision in United States v. Mackey.

Found in DMHL Volume 32 Issue 3

Ninth Circuit Finds Constitutional Right to Testify at Competency Hearing; Right Can Only Be Waived by Defendant, Not Counsel

United States v. Gillenwater, 717 F.3d 1070 (9th Cir. 2013)

The Ninth Circuit Court of Appeals held on June 17, 2013 that a defendant has a constitutional and statutory right to testify at his pretrial competency hearing and only the defendant, not his counsel, can waive that right. The Court also held that the district court must first warn the defendant that his disruptive conduct may result in his removal from the courtroom and thus the loss of his right to testify. The Court further found that denial of the defendant’s right to testify in this case was not harmless error, resulting in reversal of the district court’s decision and remanding the case for a new pre-trial competency hearing.

The defendant Charles Lee Gillenwater, II, was charged in August 2011 in the Eastern District of Washington with two counts of transmission of threatening communications and a third count of transmission of threatening communications by United States mail. Gillenwater had previously worked on a construction project at Caesar’s Palace in Las Vegas, observed what he believed to be asbestos, and began taking increasingly drastic steps to report the situation to the Occupational Safety and Health Administration (“OSHA”). Following his indictment, the district court appointed the federal defender to represent Gillenwater. After the federal defender moved to withdraw as counsel, the court appointed a private attorney to represent him. Then after receiving several letters from Gillenwater concerning the public defender and hearing from the private attorney and Gillenwater in court, the court appointed additional counsel to meet with the defendant and report whether there was a need for a competency hearing. Upon receipt of this report, the court ordered a psychological evaluation and competency hearing.

Gillenwater was transferred to a federal detention center for evaluation but was uncooperative in the evaluation process. Although unable to fully interview Gillenwater or perform psychiatric tests, the examining psychologist submitted a report based upon her clinical interviews, observations of his behavior, and a review of his legal and medical records. The psychologist diagnosed Gillenwater as suffering from a delusional disorder, persecutory type that could substantially impair his ability to assist counsel in his defense. The psychologist reported and testified at the hearing that Gillenwater described his case as a government conspiracy to silence him from reporting OSHA violations and that he believed he was the victim of “tens of thousands” of computer attacks, that he was under constant surveillance, that people from OSHA and the casino were after him, and that newspapers had been bought off from reporting his allegations. Gillenwater had also accused his attorneys of committing crimes. According to law enforcement records, Gillenwater had contacted numerous State and federal officials including a US Senator from Washington state, saying powerful people were trying to kill his staff and frame him, and that the FBI would not protect him. Gillenwater also asked his attorney to subpoena 50-plus witnesses, including Obama Administration cabinet members, so that he could take his conspiracy theory to trial.

At the competency hearing held on January 12, 2012, the government only submitted the psychologist’s report and called her as a witness. It then recommended that Gillenwater receive competency-based restoration treatment. After the government finished introducing its evidence, Gillenwater’s attorney informed the court that Gillenwater wanted to testify but that he had advised him against it, and then stated the defendant had no further evidence. During this process, Gillenwater was whispering loudly to his attorney and then interrupted his counsel calling him a criminal. When admonished by the court for interrupting the proceedings, Gillenwater continued his expletive-filled remarks, and asked to be taken out of the courtroom, stating the evidence would clear him of the diagnosis, that the judge would not be a judge much longer, and that he would wait for the Republicans to be back in charge again. The court ordered him removed from the courtroom. It then found Gillenwater did not appear to understand the charges or the court process or to be able to assist counsel in his defense and ordered him remanded to the custody of the Attorney General for 60 days.

On appeal, the Ninth Circuit reviewed Gillenwater’s contention that he had been denied his right to testify at his pre-trial competency hearing and had not waived that right as a result of his disruptive behavior. The Ninth Circuit first determined that under federal law, 18 U.S.C. § 4247(d), a defendant has the right to testify at a pretrial competency hearing. The Ninth Circuit further found that the right to testify is contained in the Fourteenth Amendment due process guarantee of the right to be heard and to offer testimony. Moreover, the Ninth Circuit found that the right to testify is also embodied in the Compulsory Process Clause of the Sixth Amendment which grants a defendant the right to call witnesses in his favor. Logically included in that right, the Court noted, is the right to testify on one’s own behalf. This right is further found in the corollary to the Fifth Amendment right against self-incrimination. If a defendant cannot be compelled to testify against himself, he must also have the right to testify. Reviewing prior Supreme Court decisions holding that an individual has the right to testify in extrajudicial proceedings, such as probation revocation hearings and hearings involving termination of welfare benefits, the Ninth Circuit went on to hold that a defendant must have an equivalent right to testify in his pre-trial competency hearing.

The Ninth Circuit then held that because a defendant’s right to testify is a personal right, it can be relinquished only by the defendant himself, and the waiver must be knowing and intentional. The Court recognized that obtaining a knowing and intentional waiver may be difficult when the defendant’s competency is in question, but it noted that defense counsel plays an important role in ensuring that the defendant understands his right to testify, that it can be waived, and the consequences of either decision. Here, the Court found that Gillenwater clearly demonstrated that he wanted to testify despite his counsel’s advice to the contrary.

The Ninth Circuit also determined that a court has no affirmative duty to inform a defendant of his right to testify, but stated it does have a duty to warn the defendant of the consequences of his disruptive behavior before it removes him from the courtroom. In this case, the court never advised Gillenwater that his behavior could lead to the loss of his right to testify. Although Gillenwater asked to be removed from the courtroom, he never expressed any desire to waive his right to testify. The court did not expressly warn Gillenwater that his removal would result in the loss of his ability to testify and therefore he never effectively waived that right.

The Ninth Circuit went on to find that where a defendant is denied a constitutional right, the court on appeal must determine whether the denial was harmless error beyond a reasonable doubt. Here, the Court found that the district court only considered a single, incomplete psychological report and Gillenwater’s conduct in the courtroom. The Ninth Circuit found other ample evidence in the psychological report that Gillenwater was very intelligent, had no criminal history, and although he was hesitant to be interviewed, was pleasant, polite, cooperative, and articulate. Based on its review of the record, the Ninth Circuit found that the denial of the right to testify was harmless error and remanded the case for a new competency hearing. The Ninth Circuit then stated that if another competency hearing is held at which Gillenwater testifies, the district court must enter an order barring the use of his testimony at his trial. Such testimony may only be used to impeach Gillenwater if he testifies at trial, but not to prove his guilt.

Found in DMHL Volume 32 Issue 3

US Supreme Court Denies Habeas Relief in Michigan’s Denial of Diminished Capacity Defense

Metrish v. Lancaster, _ U.S. _, 17 133 S.Ct. 1781, 2013 WL 2149793 (No. 12-547, May 20, 2013)

The United States Supreme Court has denied a Michigan prisoner’s petition for Writ of Habeas Corpus arguing that Michigan had erroneously prevented him from presenting evidence of his diminished capacity to a charge of first degree murder. At the time this case first came to trial in 1993, the Michigan Court of Appeals had long recognized the defense of diminished capacity to negate the mens rea or specific intent element required to support a first degree murder conviction. Two years after the first trial, the Michigan Supreme Court held that the diminished capacity defense had been abolished following Michigan’s 1975 comprehensive enactment of its statutes related to the admissibility of evidence of mental illness and intellectual disability. Upon his retrial in 2005 and on direct appeal, the Michigan courts refused to allow the defendant to present evidence of diminished capacity rejecting his argument that retroactive application of the state’s Supreme Court decision did not violate his due process rights. The Sixth Circuit granted the petitioner habeas relief. The United States Supreme Court reversed finding that the state court decisions did not result in an unreasonable application of clearly established federal law as embodied in Supreme Court decisions. The Opinion is available on the Court’s website at: http://www.supremecourt.gov/opinions/12pdf/12-547_0pm1.pdf.

In April 1993, Burt Lancaster, a former police officer with a long history of mental illness, shot and killed his girlfriend in a shopping center parking lot. Lancaster was charged with first degree murder and possession of a firearm to commit a felony. At his jury trial in 1994, Lancaster raised the insanity and diminished capacity defenses. At that time, the Michigan Court of Appeals in a line of cases had permitted legally sane defendants to present evidence of mental abnormality to negate the specific intent or mens rea required to commit a crime. Even though he was allowed to present this evidence, the jury convicted him of both charges. Lancaster later obtained federal habeas corpus relief on different grounds because the prosecutor at his first trial had erroneously exercised a race-based preemptory challenge to a potential juror and was awarded a new trial.

Prior to his new trial, however, the Michigan Supreme Court determined that the Michigan legislature had enacted a comprehensive legislative scheme in 1975 establishing the requirements for introducing evidence related to a defense based upon mental illness or intellectual disability. People v. Carpenter, 627 N.W.2d 276 (Mich. 2001). That scheme essentially adopted the M’Naughten rule for asserting an insanity defense. It also required a 30- day notice of intent to use the defense and a court-ordered psychiatric examination. In addition, the legislature created a verdict of “guilty but mentally ill” for defendants who suffered from mental illness but did not satisfy the legal definition of insanity. Such defendants would be provided with treatment but would not be exempt from the sentencing provisions applicable to other criminal defendants. Although the legislation did not specifically address the diminished capacity defense, the Michigan Supreme Court found that by creating such a comprehensive statutory scheme, the diminished capacity defense was superseded by that scheme.

Upon retrial in 2005, the trial court refused to allow Lancaster to assert the diminished capacity defense based upon the decision in Carpenter and he was again found guilty of first degree murder and sentenced to life in prison. The Michigan appellate courts upheld the conviction and the United States District Court denied him habeas relief. On appeal, the Sixth Circuit Court of Appeals reversed holding that the Michigan Supreme Court’s decision in Carpenter was unforeseeable because of 1) the Michigan Court of Appeals’ consistent application of the diminished capacity defense, 2) the Michigan Supreme Court’s repeated references in dicta to the defense, and 3) the Michigan State Bar’s use of the defense in pattern jury instructions.

After granting the petition for Writ of Certiorari, the United States Supreme Court reversed the Sixth Circuit. Writing for a unanimous Court, Justice Ginsburg recognized that a state prisoner has a very high standard to meet to obtain habeas corpus relief from a federal court. To obtain such relief, the challenged court ruling must have unreasonably applied federal law clearly established in United States Supreme Court decisions. The Court declined to apply Bouie v. City of Columbia, 378 U.S. 347 (1964), as urged by Lancaster, a case in which the Court had preciously held that due process required state criminal statutes must give fair warning of the conduct they prohibit. In Bouie, African-American petitioners had been convicted of trespass under South Carolina law after they refused to comply with orders to leave a drug store’s restaurant, which was reserved for white customers. Unlike this case, the South Carolina Supreme Court had unexpectedly expanded narrow and precise statutory language that did not cover the defendants’ conduct.

Instead the Court relied upon Rogers v. Tennessee, 532 U.S. 451 (2001), a case that upheld the Tennessee Supreme Court’s retroactive abolishment of the year and a day rule, a common law rule that barred a murder conviction unless the victim died within a year and a day of the act. In Rogers, the Court held that the retroactive application of the decision did not violate due process. The Court recognized that the diminished capacity defense is not an outdated relic of the common law as is the year and a day rule. To the contrary, the Court observed that the Model Penal Code sets out a version of the defense whenever evidence may establish a defendant does not have the state of mind that is necessary to establish an element of the offense.

In addition, the American Bar Association approved criminal justice guidelines in 1993 that favor the admissibility of mental health evidence to negate mens rea, and a majority of States, including Virginia, allow such evidence in certain circumstances. Nevertheless, the Court held that it has never found a due process violation where a state supreme court “squarely addressing a particular issued for the first time, rejected a consistent line of lower court decisions based on the supreme court’s reasonable interpretation of the language of a controlling statute.” The Supreme Court therefore reversed the Sixth Circuit decision and denied Lancaster habeas relief.

Found in DMHL Volume 32 Issue 3

Washington Supreme Court Finds Competency Evaluation Open to Public When It Becomes Court Record

State v. Chen, 309 P.3d 410 (Wash. 2013)

Under Washington law, competency evaluations are confidential and available only to certain specified individuals with a need to access the information. The Washington Supreme Court held on September 5, 2013 that the State constitutional requirement that all cases be administered openly supersedes that law. A court may seal a competency evaluation only when it makes an individualized finding that factors enumerated in a Washington Supreme Court case, Seattle Times Co. v. Ishikawa, 97 Wn.2d 30, 640 P.2d 716 (1982) weigh in favor of sealing.

Defendant Louis Chen was charged with two counts of aggravated murder that occurred in August 2011. Chen’s attorney presented mitigation information to discourage the State from seeking the death penalty. Part of this information was an opinion from a psychiatrist that Chen was not competent to stand trial. As a result, the trial court ordered that Chen be evaluated by doctors at Washington’s Western State Hospital. After receipt of the evaluation, the court found Chen competent to stand trial. Chen moved to seal the competency evaluation, or in the alternative, to redact certain information. Under Washington law,

…all records and reports made pursuant to this chapter, shall be made available only upon request, to the committed person, to his or her attorney, to his or her personal physician, to the supervising community corrections officer, to the prosecuting attorney, to the court, to the protection and advocacy agency, or other expert or professional person who, upon proper showing demonstrates a need for access to such records.

RCW 10.77.210(1).

The trial court denied Chen’s motion to seal the evaluation, applying the Ishikawa factors, but did redact certain information contained in the report. Under Ishikawa, anyone seeking closure of court proceedings must make some showing of a compelling interest, and where the interest is based on a right other than the accused’s right to a fair trial, that showing must demonstrate a “serious and imminent threat” to that right. Anyone present must be given an opportunity to object to the closure. The method of closure must be the least restrictive means available for protecting the threatened interest. A television station was in the courtroom and objected to the motion to seal. Direct discretionary review of this decision was granted and during the pendency of the appeal, the trial court stayed its order and sealed the entire evaluation pending review.

On appeal, Chen first argued that if competency evaluations are subject to openness, the statute would be rendered meaningless. The Court held, however, that the statute applies until the competency evaluation becomes a court record, at which point it becomes open to the public. Chen also argued that important privacy issues are at stake and that public access could taint the jury pool. The Court found that these are important considerations, but they can be adequately addressed as part of a motion to seal. The Court found that competency determinations are an important turning point in the criminal process and the idea of a public check on the judicial process is especially important when competency is at issue. Having found that the statute conflicted with the State constitutional requirement of openness, and that Chen was seeking a blanket exclusion for all competency evaluations, the Court held that the trial court had not abused its discretion in refusing to seal the evaluation and to redact only certain portions of the report.

Found in DMHL Volume 32 Issue 4

Texas Appellate Court Finds Trial Court Lacks Authority to Order Incompetent Inmate to Be Involuntarily Medicated to Restore His Competency to Be Executed

Staley v. Texas, _ S.W.3d _, 2013 WL 4820128 (Tex.Crim.App. 2013)

The Texas Court of Criminal Appeals held on September 13, 2013 that a trial court had no authority to order a mentally ill inmate, who had previously been found incompetent to be executed to be medicated under the State’s competency-to-be-executed statute and therefore vacated the execution order.

In 1991, Steven Kenneth Staley was convicted of capital murder when he and two others rounded up a group of employees at a restaurant, threatened them with firearms, and killed the manager after taking him hostage. The trial court has since held two competency hearings finding Staley incompetent to be executed at the first hearing, and competent at the second.

A month before his scheduled execution in 2006, Staley filed a motion challenging his competence to be executed, arguing that his competence was “artificial” due to his involuntary medication. The trial court appointed two clinical forensic psychologists as experts to evaluate him. They both found that Staley suffered from paranoid schizophrenia for which he had routinely been diagnosed for 15 years, and that his condition had deteriorated over time. They reported that although he understood he was to be executed, Staley did not have a rational understanding of the reason for his execution. They further testified that Staley had been prescribed medications, mainly Haldol, through the years but that he had not consistently complied with his mediation regimen. In the months immediately preceding the competency evaluations, he had frequently refused the mediation.

One of the evaluators testified that Staley demonstrated numerous symptoms of psychosis over the years, including self-inflicted injuries, grossly neglected personal hygiene, including resting in his own urine and excrement, irregular eating and sleeping habits, and delusions of paralysis to the extent of lying in bed so long as to rub a bald spot on his head. The psychologist further described Staley’s history of spontaneously and repeatedly refusing medication, and testified that he would probably require compulsory medication for long-term control of his symptoms. He stated that good medical practice would involve medication to control his symptoms.

The other clinical psychologist also testified that Staley’s symptoms included “syntactical aphasia,” which is the nonsensical ordering of words as well as the regular use of fictitious language. He further testified that when Staley was medication compliant he showed no symptoms of decompensation, but he frequently refused medication because he denied his illness, believing it was an attempt to poison him. Based on all of the testimony, the trial court found Staley incompetent to be executed.

The following month, the State moved the trial court to order involuntary medication, arguing both the medical purpose of the medication and the State’s interest in enforcing the judgment. Staley opposed the motion arguing that the side effects of the medication were harmful, and that the medication only produced “artificial competence” and did not therefore meet the competency-to-be-executed standard under the federal or Texas constitutions or the Texas statute.

The trial court authorized the involuntary medication finding that (1) the State has a legitimate interest in enforcing the sentence that is not outweighed by the inmate’s interest in avoiding medication; (2) the medication is the least intrusive and only method of achieving competency; (3) compelling medication is in the inmate’s best medical interest because without it he will suffer “frightening delusions and general disorder within his mind” and there is no evidence he had suffered side effects from the medication; and (4) without medication he posed a danger to himself and others. Staley immediately appealed this decision to the Court of Criminal Appeals, but the Court found the involuntary medication order to be a non-appealable interlocutory order.

In 2012, the State filed a request with the trial court for a further competency examination. The court heard evidence from one of the clinical psychologists who had testified at the first hearing and another clinical psychologist, both of whom found Staley was now competent to be executed. They testified that although Staley was experiencing delusional thoughts, his symptoms were under control with about 60% compliance with Haldol and that he knew many of the details of the litigation and crime. He knew the names of the defense attorneys, prosecutors, and the victim, and that the death penalty was, in his words, to “retribute the public for a heinous crime.” He also understood the lethal injection process and described the death process as permanently going to sleep. Staley did not actually believe, however, that he would be executed because he thought his attorneys would obtain a stay. One expert testified that Staley met the competence-to-be-executed standard under the statute because he understood (1) that he was to be executed and his execution was imminent and (2) the reason for his execution. After hearing the evidence, the trial court found Staley competent to be executed, but only because of the effects of forced medication. Staley then appealed this decision to the Texas Court of Criminal Appeals arguing, among other things, that the trial court lacked authority to order his involuntary medication to restore him to competency to be executed.

On appeal, the Texas Appellate Court observed that a trial court derives its jurisdiction only from state law or the Texas Constitution, and once a conviction has been affirmed on appeal, general jurisdiction is not restored in the trial court. A trial court obtains jurisdiction post-conviction under a number of different Texas statutes, for example, to set an execution date, conduct DNA testing, or determine whether an inmate is competent to be executed. If an execution is stayed based on a determination that the inmate is incompetent, the trial court is required to order the inmate’s periodic re-evaluation by mental health experts to determine whether he is no longer incompetent to be executed. The Court held, however, that the statute does not convey the authority on the trial court to order involuntary medication to restore the inmate to competency to be executed. The Court rejected the State’s argument that the trial court had inherent or implied authority to order involuntary medication or that such a lack of authority would produce an absurd result in cases such as this.

By contrast, Texas statutes permit the involuntary medication of people who are involuntarily committed, or are incompetent and awaiting trial. The Court noted that in Texas this process involves an administrative hearing, not a judicial hearing, before a non-treating psychiatrist. Trial courts may also order a defendant to be forcibly medicated when under court order to receive inpatient mental health services or to be restored to competency to stand trial, but not to be executed.

Because the evidence demonstrated that Staley would have been incompetent to be executed but for the trial court’s involuntary medication order, the Court held that he did not meet the competency-to-be-executed definition under the Texas statute or the Texas or federal constitutions. The trial court’s unauthorized order was the sole cause of the transformation of evidence from supporting a finding of incompetence to one of competence. The Court therefore vacated the finding of competence and remanded the case to the trial court for periodic reexaminations of Staley’s competence to be executed.

Found in DMHL Volume 32 Issue 4

Fifth Circuit Holds Capital Defendant Not Entitled to All Expert Funding Requested; Was Competent-to-Be-Executed; Edwards Decision on State’s Right to Deny Self-Representation Not Retroactive

Panetti v. Stephens, 727 F.3d 398 (5th Cir. 2013.)

The Fifth Circuit Court of Appeals upheld the death penalty on August 21, 2013 for a mentally ill inmate alleging incompetence-to-be-executed, finding the district court’s decision to deny funding for additional expert assistance and testing was not an abuse of discretion. The Fifth Circuit also held that the district court’s decision weighed all of the evidence, including the inmate’s secretly recorded conversations with family, and was therefore not clearly erroneous. The Court further held that the United States Supreme Court case of Indiana v. Edwards, 554 U.S. 164 (2008), holding that the State may prohibit a mentally ill inmate found competent to stand trial from representing himself at trial had no retroactive application in federal habeas corpus proceedings.

In 1992, Scott Louis Panetti shot his estranged wife’s parents at close range, killing them and spraying his wife and three-year-old daughter with their blood. Panetti demanded to represent himself at trial although he had a long history of schizophrenia, and in spite of the trial judge’s pleas to accept counsel. His only defense was insanity. His appointed standby counsel described his self-representation as bizarre and his trial a farce and mockery of self-representation. The jury convicted him of capital murder and sentenced him to death. The conviction and sentence were upheld on direct appeal and collateral review.

In October 2003, the trial court set an execution date and Panetti filed a motion alleging for the first time that he was incompetent-to-be-executed. The trial court rejected the motion without a hearing. Texas law required Panetti to make a “substantial showing of incompetency” before entitling him to court-appointed experts. On federal habeas review, Panetti submitted additional evidence of mental illness and the district court stayed the execution to permit the state trial court to consider the renewed motion in light of the supplemental evidence. In February 2004, the state court appointed a psychiatrist and a clinical psychologist to examine Panetti, implicitly finding he had made a substantial showing of incompetency. These experts filed a joint report finding Panetti competent-to-be-executed. Without holding a hearing or ruling on Panetti’s motion to appoint him his own experts, the trial court found Panetti competent-to-be-executed.

Panetti then returned to federal court arguing that Texas’ failure to appoint him mental health experts and provide a hearing violated his due process rights under Ford v. Wainright, 477 U.S. 399 (1986). Ford held that denying a prisoner the right to present and rebut evidence in a competency-to-be-executed proceeding violated due process. The district court agreed and also found that such a denial by the state court was not entitled to deference under the Antiterrorism and Effective Death Penalty Act. Panetti’s experts then testified that he understood the reason for his execution – the murder of his in-laws, but his delusions caused him to believe Texas was in league with the forces of evil and sought to prevent him from preaching the Gospel. The State’s experts agreed Panetti was mentally ill, but his behavior was attributed to malingering. After hearing the evidence, the district court found that Panetti’s delusional belief system prevented him from rationally appreciating the connection between his crimes and his execution. But the district court found Panetti competent to be executed because the Fifth Circuit standard at that time was that the prisoner only needed to know the fact of his impending execution and the reason for it. The Fifth Circuit affirmed the district court decision and Panetti petitioned the United States Supreme Court for review.

In 2007, the United States Supreme Court granted Panetti’s petition for certiorari and reversed, finding the Fifth Circuit’s standard for competency-to-be-executed too restrictive. Declining, to set out a standard, the Supreme Court remanded the case requiring the district court to determine in a more definitive manner the nature and severity of Panetti’s mental health problems and whether his delusions impaired his concept of reality to the extent that he did not have a rational understanding of the reason for the execution. Panetti v. Quarterman, 551 U.S. 930 (2007).

On remand, the defense hired three experts, a clinical neurologist, a forensic psychiatrist and a forensic psychologist who had examined Panetti for the original hearing in 2004. These experts evaluated Panetti for a combined total of over 15 hours and administered a battery of tests designed to detect the likelihood of malingering. The district court authorized $9000 to pay the experts, but rejected his requests for additional funding. These experts all diagnosed Panetti with schizophrenia, although the psychologist who had examined him previously found Panetti had markedly improved since his 2004 examination. The other two experts testified that Panetti suffered from a genuine delusion that he was on death row to preach the Gospel and save souls. The defense also called two death row inmates who testified that Panetti preached incessantly in his cell and in the day room even though it irritated other inmates.

Texas presented testimony from a forensic psychologist and an expert psychiatrist and neurologist. Both testified that Panetti was partially fabricating his symptoms to thwart attempts to administer tests to detect malingering. The psychiatrist also doubted whether he was suffering from any form of mental illness, and was emphatic that Panetti had a rational understanding between his crime and execution because of his repeated assertions that he was unjustly convicted despite his insanity and that God had forgiven his guilt. Texas also called three correctional guards as witnesses who testified Panetti was never a problem; was generally wellbehaved, but would often have some religious statement to make; that the preaching was well thought out and the same as you would hear at church; and that some guards would assign Panetti to a cell to get revenge on an inmate because they knew his constant preaching would irritate him.

Texas also presented secret recordings of his conversations with his family. The tapes indicated that while Panetti did quote scripture and make religious comments, he did not rant or preach, and the conversations involved extended discussion about the trial judge’s corruptness and ineptitude. The district court’s summary of the tapes reflects that Panetti at no time became irrational, tangential or pressured in his speech. His comments about his legal proceedings reflected a fairly sophisticated understanding of his circumstances.

After hearing all of this evidence, the district court found that Panetti was seriously mentally ill and suffered from paranoid delusions of some type. The court also determined that Panetti was exaggerating some of his symptoms to avoid execution, stating that Panetti demonstrated a fairly sophisticated understanding of his case and that his refusal to cooperate with State experts contrasted with his treatment of his own experts. The district court then determined that Panetti had both a factual and rational understanding of his crime, his impending death and the causal retributive connection between the two.

On appeal, the Fifth Circuit first determined that the district court did not abuse its discretion in refusing to provide additional funding to permit his experts to review the secret recordings and to obtain a PET scan to detect malingering. The Fifth Circuit found that the district court authorized $9000 to fund an expert team to assist Panetti in presenting his competency evidence and they were able to review a large number of the secret recordings. The request for a PET scan also violated the court’s scheduling order. Although the Supreme Court’s decisions in Ford and Panetti established a constitutional right to expert assistance in Eighth Amendment competency-to-be-executed hearings, the Court held the cases merely entitle the inmate to an opportunity to present his own expert testimony before a neutral decision maker. The decisions do not require the court to provide all of the expert assistance the inmate requests.

The Fifth Circuit also agreed that the Supreme Court’s remand required a “rational understanding” test for Eighth Amendment competency-to-be-executed proceedings, but disagreed with the district court determination, finding that the test is not the same as the Dusky standard applied in competency to stand trial situations. The Eighth Amendment standard arises out of the retributive value of executing a person who has no comprehension of why he is being executed and the abhorrence of civilized societies to kill someone who has no capacity to come to grips with his own conscience or deity. Nonetheless, the Fifth Circuit agreed that the district court applied the correct rational understanding analysis in finding Panetti had both a factual and rational understanding of his crime, his impending death and the causal retributive connection between the two, based especially upon Panetti’s rationally articulated position that his punishment was unjustified because of his insanity at the time of his offense. The Fifth Circuit then found that the expert testimony was conflicting and that the district court’s finding of competency was therefore not clearly erroneous. The Court also found that the secret recordings generally corroborated the testimony of the State’s experts and that Panetti actually understood the reason for his punishment.

Finally, Panetti raised for the first time before the Fifth Circuit the issue that the State should not have permitted him to represent himself at trial. At the time of his trial, Panetti had been found competent to stand trial and then insisted on exercising his right of selfrepresentation. The United States Supreme Court cases of Faretta v. California, 422 U.S. 806 (1975), holding that defendants have a Sixth Amendment right to represent themselves, and Godinez v. Moran, 509 U.S. 389 (1993), suggesting that this right was absolute even if invoked by a severely mentally ill defendant, had been decided at the time of Panetti’s trial. The Supreme Court later held in Indiana v. Edwards, 554 U.S. 164 (2008), after Panetti’s trial, that the right of self-representation was not absolute and the State could insist that an attorney be appointed to represent a mentally ill defendant even though he had been found competent to stand trial.

The Fifth Circuit found, however, that this decision had no retroactive application to habeas petitions. In order to apply a new rule of constitutional law retroactively to federal habeas proceedings, the new rule must be a watershed rule of criminal procedure implicating the fundamental fairness and accuracy of the criminal proceedings. The Court found that the right of the State to impose representation on a mentally ill defendant did not effect a sea change in criminal procedure. The Edwards decision also only applies in the exceptional situation where the defendant is competent to stand trial but so severely mentally ill that his self-representation threatens an improper conviction or sentence. Furthermore, Edwards is only permissive, allowing the state to insist on counsel but not requiring that it do so. The Court held that its application was therefore not retroactive.

Found in DMHL Volume 32 Issue 4

Tenth Circuit Requires Treatment Plan with Medications and Maximum Dosages before Authorizing Involuntary Medication of Incompetent Defendant

United States v. Chavez, 734 F.3d 1247 (10th Cir. 2013)

The Tenth Circuit Court of Appeals overturned on November 13, 2013, the district court’s order authorizing treatment of an incompetent defendant with antipsychotic medication over his objection. Following the precedent of three other circuits, the Tenth Circuit held that the government must submit a treatment plan containing the proposed medications and maximum dosages before the trial court can determine whether the second and fourth prongs required under United States v. Sell, 539 U.S. 166 (2003), have been met in order to justify an involuntary medication order. In order to authorize medication of an incompetent defendant under Sell, the government must establish 1) that important governmental interests are at stake, 2) the involuntary medication will significantly further those interests, 3) the involuntary medication is necessary to further those interests, and 4) the administration of the mediation is medically appropriate and in the defendant’s best medical interests.

Reydecel Chavez, a Mexican citizen, was arrested in New Mexico and charged with being a felon in possession of a firearm, being an illegal alien in possession of a firearm, and reentry into the United States as a removed alien. Soon after he was charged, both his attorney and the government agreed that Chavez should be evaluated for his competency to stand trial. The district court committed him to the Bureau of Prisons medical center in Springfield, Missouri for a competency determination. The psychologist performing the evaluation reported that Chavez was diagnosed with paranoid schizophrenia and was not competent to stand trial. He also reported that Chavez was not a danger to himself or others while in custody and could likely be restored to competency with antipsychotic medication, which he was refusing.

At the competency hearing, the district court found Chavez incompetent to stand trial, and at the court’s suggestion, the government filed a motion to require him to undergo treatment with medication over his objection. The same evaluator testified in general terms as to the treatment Chavez would likely receive, but the government presented no individualized treatment plan. The evaluator testified that an individualized treatment plan would be prepared for Chavez only after involuntary treatment was authorized by the court. He also testified that as a psychologist he could not prescribe medication for Chavez, but that the “typical” treatment plan would involve injection with Haldol. Side effects could be addressed with a change in medication or administration of drugs specifically designed to treat them. He further testified that three-fourths of defendants treated with antipsychotic medications are successfully restored to competency. Following this testimony, and over Chavez’s objection, the district court found that a specific treatment plan was not necessary to meet the Sell requirements and ordered his treatment over objection, requiring only a status report in about six weeks.

On appeal, the Tenth Circuit found that the first two prongs of the Sell test were purely issues of law for the appellate court to decide de novo, but the third and fourth prongs were factual determinations that the appellate court would reverse only if they were clearly erroneous with no evidence in the record to support them. The Tenth Circuit then analyzed the evidence supporting the Sell requirements and agreed with Chavez’s arguments that without a specific treatment plan identifying which medications would be administered to him and at what doses, the district court had insufficient information to make the required findings. The Court of Appeals found that the need for a high level of detail is plainly contemplated by the Sell case. Without evidence in the record that a psychiatrist who will be prescribing the drugs solely to render him competent to stand trial, the court cannot ensure as a legal matter under the second Sell prong that the administration of the drugs will be substantially unlikely to produce side effects that will interfere with Chavez’s ability to assist his attorney in presenting a defense. Also, without knowing which drugs the government might administer and at what dosage, the court has no evidence upon which it can determine under the fourth prong whether the treatment will be medically appropriate for Chavez. In addition, the court’s order sets no meaningful limits on the government’s discretion in treating Chavez and is so open-ended that it would give treatment staff carte blanche to experiment with what might be dangerous drugs or dangerously high dosages of drugs.

In making these findings, the Tenth Circuit followed the decisions from three other circuits. The Ninth Circuit held in United States v. Hernandez-Vasquez, 513 F.3d 908 (9th Cir. 2007), that to pass muster under Sell, 1) the district court’s order must identify the specific medication or range of medications the physicians are permitted to use, 2) the maximum dosages, and 3) the duration of time the involuntary medication may continue before requiring a report back to the court. Similarly, the Fourth Circuit held in United States v. Evans, 404 F.3d 227 (4th Cir. 2005), that the government must set forth the particular medication, including the dosage. The Sixth Circuit in United States v. Green, 532 F.3d 538 (6th Cir. 2008), also upheld a specific treatment plan that set forth the specific medications, alternative means of injecting it, the specific dosage, and the potential side-effects.

In following these cases, the Tenth Circuit decided that a balance must be struck between the judicial oversight needed to protect the defendant’s constitutional rights and the need of medical staff to retain flexibility in providing effective treatment. The Court then held that a court may approve a treatment plan as long as all drugs that might be administered to a defendant and their maximum dosages are specified. In so doing, the Tenth Circuit found that the district court’s order lacked sufficient information to determine whether the second and fourth requirements under Sell were met, reversed the order and remanded the case for further proceedings.

Found in DMHL Volume 33 Issue 1

Ninth Circuit Holds Expert in Competency Evaluation May Testify As to Diminished Capacity Defense

United States v. Christian, _F.3d _, 2014 WL 1491887 (9th Cir. Apr. 17, 2014)

The Ninth Circuit Court of Appeals held on April 17, 2014, that the district court abused its discretion in refusing to permit an expert psychologist who had completed a competency evaluation of the defendant to testify in support of the defendant’s diminished capacity defense. The Court of Appeals determined that the district court had focused in error on the different legal standards related to competency to stand trial and diminished capacity rather than focusing on whether the substance of the expert’s testimony would have assisted the jury in deciding whether the defendant could form the requisite intent to commit the crimes charged.

Eric Leon Christian was convicted of two counts of sending threatening emails through interstate commerce. In May 2009, he first emailed the then Chief of the North Las Vegas Police Department asking for assistance in retrieving his car, which had been repossessed several months earlier. When the police chief responded that the police could not assist him, he sent an email containing several threats of violence, including “I will have to kill to retrieve my stolen and items [sic] if you do not return them” and “This communication is protected by the First Amendment and my undying declaration of ridding the earth of terrorist, [sic] who take away Constitutional Rights like YOU and the thief who has my car.” Christian also emailed threats to the then chief deputy city attorney and the chief prosecutor for North Las Vegas. He had requested copies of case files for two cases, neither of which had been prosecuted by the attorney or the city attorney’s office. Christian followed with an email threatening to “get a mob together and start a civil war” to kill a state court judge or the attorney himself unless the attorney “g[o]t the Writ of Habeas Corpus out of the way.”

In an unrelated state court proceeding, a psychologist had evaluated Christian’s competency to stand trial. The psychologist interviewed Christian for about an hour, following an outline designed to assess the core competencies related to a defendant’s ability to assist in his defense. Christian was hostile and uncooperative during the interview, and the psychologist assessed his competence level in all areas to be inadequate. He determined that Christian could not communicate relevant information to his attorney, make rational decisions about plea bargaining or asserting a mental illness defense, or behave appropriately in the courtroom. The psychologist diagnosed Christian with psychosis, not otherwise specified, probably delusional or paranoid; personality disorder, not otherwise specified; and probable learning disabilities, not otherwise specified. He also believed Christian was at high risk for homicidal behaviors and recommended his transfer to another facility for treatment and medication management.

Prior to trial on these charges, Christian gave notice of his intent to call the psychologist as an expert witness. His sole defense to the charges was diminished capacity in which he argued that he was incapable of forming the specific intent required by the charge, in this case, the specific intent to threaten. The police chief and attorney both testified for the prosecution that they had felt threatened by the emails and had taken precautions to protect themselves and others. The police chief also testified that he thought Christian was very upset and “very disturbed.” At the close of the government’s case, Christian’s attorney informed the district court that he intended to call the psychologist to testify regarding Christian’s diminished capacity defense. Without conducting a voir dire of the psychologist to determine the substance of his testimony or to permit counsel to proffer what his specific testimony would be, the district court determined that because the legal standards for competency to stand trial and the diminished capacity defense, which concerns “whether the defendant has the ability to attain the culpable state of mind which defines the crime,” the expert could not testify. Christian’s attorney then rested his case without presenting any other evidence or calling Christian to testify. He then asked for a diminished capacity instruction to the jury, which the district court denied because there was no evidence in the record that would support such an instruction. The jury convicted Christian on both counts.

Christian appealed to the Ninth Circuit Court of Appeals arguing that the district court had abused its discretion by refusing to let him introduce expert testimony in support of his diminished capacity defense and refusing his diminished capacity instruction. The Court of Appeals agreed with Christian on the district court’s refusal to admit the expert testimony, finding that the district court had focused exclusively on the different legal standards. Instead the court should have determined whether the testimony regarding he psychologist’s observations and diagnoses of Christian would have been relevant and helpful to the jury in evaluating his diminished capacity defense. The Court then decided that the failure to allow the expert testimony was not harmless error and a new trial was necessary. Because the district court had not conducted a voir dire of the expert, the district court on remand must first do so, assessing whether the testimony would in fact be relevant and would assist the jury in assessing the diminished capacity defense. Once the district court does this, and if it then determines the evidence is relevant and would be of assistance to the jury, it should allow the testimony.

The Ninth Circuit rejected Christian’s argument that the district court erred in not giving the jury a diminished capacity instruction. He had argued that the threats in the emails themselves were inherently irrational and were sufficient to warrant a diminished capacity instruction. He also argued that even the police chief believed him to be “very disturbed.” The Ninth Circuit found that these arguments alone were insufficient to require a diminished capacity instruction, but if there had been any evidence in the record, the instruction should have been given. The Appellate Court then held, however, that if any evidence on remand, expert or otherwise, supports a link between Christian’s mental illness and his ability to form the intent to threaten, then the instruction must be given. The Ninth Circuit vacated the conviction and remanded the case to the district court for a new trial.

Found in DMHL Volume 33 Issue 2

Competency to Stand Trial/Restoration of Competency

Sell criteria for involuntary treatment to restore competency apply to sentencing phase

United States v. Cruz, 757 F.3d 372 (3d Cir. 2014) cert. denied, No. 14-7512, 2015 WL 133477 (U.S. Jan. 12, 2015)

Cruz was arrested and convicted on two counts of threatening a federal law enforcement officer. After the court received the pre-sentence investigation report, the prosecution successfully moved for a determination of competency. A Federal Bureau of Prisons forensic psychologist concluded that Cruz was mentally incompetent and suffered from schizophrenic disorder, bipolar type. After a hearing, the court concluded that Cruz was incompetent and found that he could not proceed with sentencing.

A second report concurred with the diagnosis, noted Cruz’s ongoing refusal to take antipsychotic medication recommended by BOP personnel, concluded that without medication Cruz would remain incompetent, and stated that “there is a substantial probability that [his] competency can be restored with a period of forced medication.” The prosecution obtained an order authorizing the BOP to medicate Cruz against his will.

On appeal, the issue was whether “the Government, pursuant to the Supreme Court's decision in Sell v. United States, 539 U.S. 166, 123 S.Ct. 2174, 156 L.Ed.2d 197 (2003), can have a sufficiently important interest in forcibly medicating a defendant to restore his mental competency and render him fit to proceed with sentencing.” In affirming the decision of the federal district court, the Third Circuit held that the government could have a sufficiently important interest in sentencing a defendant for serious crimes to justify involuntary medication. Relying on the stated concern in Sell that “memories may fade or evidence may be lost,” the Third Circuit held the same concern applies with equal force in the sentencing context (the guilt phase was at issue in Sell) because it means that it may be “difficult or impossible to sentence a defendant who regains competence after years of commitment.” Additionally, while it may be cognizable that some crimes are not “serious” enough to justify forcible medication at the sentencing stage, Cruz’s offense was certainly “serious” enough.

The United States Supreme Court denied certiorari in January 2015.

Found in DMHL Volume 34 Issue 1

Competency to waive counsel (Colorado)

Colorado’s state-developed process for assessing competency meets constitutional requirements and does not require adoption of process set out by the U.S. Supreme Court in Indiana v. Edwards

People v. Davis, 2015 CO 36 (Colo. 2015). (This opinion has not been released for publication in the permanent law reports and until it is released, it is subject to revision or withdrawal.)

Rashaim Davis was convicted in a Colorado state court of possession and distribution of a controlled substance and various related charges. Prior to trial, Davis informed the court that he wanted to represent himself. During a pretrial colloquy, Davis told the trial court that he was taking an antidepressant, Wellbutrin, for “bipolarism” and “mental condition as far as…not trusting people,” but that the Wellbutrin did not completely control the paranoia that had led to his mistrust of his court-appointed lawyers. The trial court found that Davis was unable to voluntarily, knowingly, and intelligently waive his right to counsel. The court of appeals reversed the trial court’s order denying Davis’s request to proceed pro se, proscribing a new standard for a criminal defendant’s competency to waive the right to trial counsel, relying on the United States Supreme Court’s decision in Indiana v. Edwards, 554 U.S. 164 (2008).

The Colorado Supreme Court reversed the court of appeals, holding that Colorado’s “existing two-part, totality-of-the-circumstances analysis to determine whether a defendant has validly waived the right to counsel affords trial courts sufficient discretion to consider a defendant's mental illness.” The Colorado Supreme Court noted that state law already requires that a waiver of the right to counsel be both “voluntary” and “knowing and intelligent,” and that “mental illness might prevent him from broadly understanding the charges, punishments, defenses, and other essential facts of the case.” Thus, a trial court could “consider the defendant’s mental illness during its totality-ofthe-circumstances-analysis.” Additionally, the Colorado Supreme Court held that Colorado law does not require an Edwards standard because it already provides “what the Supreme Court sought in Edwards: an analytical scheme that appropriately considers whether mental illness should prevent the defendant from representing himself at trial.”

Found in DMHL Volume 34 Issue 2

Competence to enter into plea agreement

District court abused its discretion in not sua sponte ordering competency evaluation based upon medical evidence of incompetence introduced for the defendant’s sentencing hearing

U.S. v. Wingo, 2015 WL 3698157 (11th Cir. 2015)

Andrew Wingo was a defendant in a complex securities fraud case, and was represented by counsel. Wingo made only brief appearances before the court, and ultimately entered into a plea agreement in which he pleaded guilty to just one of the numerous charges against him. Some evidence of Wingo’s health concerns came to light during early proceedings (e.g., bond revocation hearing), but neither his attorney nor the government raised any concerns about Wingo’s competence at the plea hearing. The issue of Wingo’s mental capacity was not raised until the sentencing phase approximately six months later, when Wingo’s attorney requested a reduced sentence based upon diagnoses of dementia and other cognitive impairments. The pre-sentence report from the government also noted Wingo’s cognitive impairments. The court at sentencing noted the medical information submitted regarding Wingo’s condition, but determined that this should not affect the length of sentence. Wingo appealed, arguing that the court had both a statutory and a constitutional duty to order a competency hearing sua sponte because there was reasonable cause to doubt his competence.

The Eleventh Circuit noted that in a prior case (Tiller v. Esposito, 911 F.2d 575, 576 (1990)) it had identified three factors to be considered in determining whether information establishes a “bona fide doubt regarding the defendant's competence.” After a detailed review of the evidence submitted prior to the sentencing hearing, the Eleventh Circuit found that the evidence was sufficient to create “reasonable cause to believe that Wingo was incompetent to proceed to trial or to plead guilty.” It found that the district court had abused its discretion in failing to sua sponte order a competency hearing, and remanded the case to the district court to determine “whether Wingo's competency can be evaluated nunc pro tunc, and if so, for an assessment of his competency at the time of his guilty plea and sentencing.” The Eleventh Circuit stated that if such evaluation were to find Wingo was incompetent at the time of the plea agreement, or if such evaluation is not possible, Wingo's conviction and sentence must be vacated, with the government having the right to try him if he becomes competent. If the evaluation were to find Wingo was competent, his conviction and sentence must be affirmed.

Found in DMHL Volume 34 Issue 2

Restoration of competency through forcible medication

United States v. Watson, 793 F.3d 416 (4th Cir. 2015)

Sell standard requires government to make a particularized showing by clear and convincing evidence that proposed treatment is substantially likely to restore defendant to competency

Background: The government requested court authorization to medicate defendant involuntarily in order to restore him to competency to stand trial. The district court granted the request finding that the government had met the Sell requirements8 by clear and convincing evidence. Defendant appealed, claiming that the government failed to satisfy (1) the first Sell prong because the “important governmental interest at stake” in prosecuting Watson was mitigated by the fact that Watson had the “possibility” of an affirmative defense of “not guilty by reason of insanity” and (2) the second Sell prong because the government did not prove that the proposed forced medication was substantially likely to render Watson competent to stand trial.

Holdings: In a 2-1 panel decision, the Court reversed the District Court, holding that the record showed that the government not only failed to meet the second Sell prong, but could not meet it, and it dismissed the matter, without remand to the District Court for further hearing.

Notable Points: Emphasizing the importance of the liberty interest at stake, the intrusiveness of the act of forcible medication, and the burden of “clear and convincing evidence” that the government had to meet, the Court ruled that the evidence in the record failed to show that the forced medication proposed for Watson was substantially likely to restore Watson to competency. The Court, noting that Watson had a “rare” delusional disorder, concluded that the government’s expert, and the research relied upon by that expert, addressed the efficacy of the proposed forced medication for people suffering from psychosis in general and that the government failed to address the medication’s likely effect on this defendant in particular.

Found in DMHL Volume 34 Issue 3

Due process and capacity determinations in immigration proceedings

Diop v. Lynch, 807 F.3d 70 (4th Cir. 2015)

Immigration judge’s inquiry into appellant’s mental competence to participate in removal proceedings met due process requirements, and resultant denial of appellant’s request for further delay to obtain a mental health evaluation did not violate due process and is upheld on appeal

Background: Madiagne Diop, an alien and native of Senegal, who was diagnosed with psychosis. Following an arrest related to a psychotic episode at his workplace, Diop appeared before an immigration judge in Baltimore, Maryland five times between November 2012 and May 2013. At a hearing in December 2012, the immigration judge questioned Diop regarding his mental health and competency and found Diop competent to participate in the removal proceedings. In April 2013, Diop moved to administratively close or continue proceedings pending the passage of an immigration reform bill in Congress. The immigration judge refused and ordered Diop’s removal if he would not voluntarily depart. On June 6, 2013, Diop filed an appeal to the BIA arguing that the immigration judge should have administratively closed or continued the case in order to allow him to receive a psychological evaluation. The BIA found no clear error in the immigration judge’s determination of Diop’s competency, and Diop petitioned for review by the Fourth Circuit.

Holdings: The Fourth Circuit Court of Appeals rejected Diop’s procedural due process claim. It held that Diop’s procedural due process rights were satisfied by the immigration judge’s cautionary measure of assessing Diop’s competency in a separate hearing and “inquiring specifically about his mental health and ability to communicate with counsel.” Because competency has “long been considered an issue of fact,” the immigration judge’s finding of competency was reviewed under a “substantial evidence standard”: findings of competency are treated as conclusive unless the evidence presented “was such that any reasonable adjudicator would have been compelled to conclude the contrary.”

Notable Points:

The process for addressing competency in removal proceedings: The BIA stated that the immigration judge should start from a presumption of competency and that if there are no indicia of incompetency, the inquiry ends. It established a competency standard in Matter of M-A-M requiring (1) "rational and factual understanding of the nature and object of the proceedings," (2) ability to "consult with the attorney or representative if there is one," and (3) "a reasonable opportunity to examine and present evidence and cross-examine witnesses."

Found in Found in DMHL Volume 34, Issue 4

Treatment over Objection to Restore Defendant to Competency to Stand Trial

U.S. v. Onuoha, 820 F.3d 1049 (9th Cir. 2016)

Ninth Circuit vacates and remands district court order authorizing treatment, finding government failed to show proposed treatment is in the defendant’s best medical interests.

Background: From 2004 to 2012, Onuoha served in the National Guard. From 2006 to September 2013, when he resigned, Onuoha worked as a Transportation Security Administration (TSA) screener at LAX Airport. A few hours after resigning, Onuoha returned to TSA headquarters at LAX and left a note for a former supervisor who had been involved in a suspension of Onuoha earlier that summer. The government alleged that Onuoha then made calls to a TSA checkpoint and to the LAX police department alluding to sending a message to America and the world and telling them to evacuate LAX. TSA headquarters was evacuated. Law enforcement officials went to Onuoha’s apartment and discovered that all of his belongings had been removed and all that remained was a large note reading “09/11/2013 THERE WILL BE FIRE! FEAR! FEAR! FEAR!” Later that day, Onuoha called LAX police and told them that he was at a church in Riverside, CA. He told police that he did not intend to make a bomb threat or injure anyone and that he only wanted to deliver a message. Onuoha waited at the church until he was apprehended.

Onuoha’s defense counsel submitted a report that Onuoha suffered from paranoid schizophrenia and planned to raise a diminished-capacity defense. The government requested a competency evaluation, and Onuoha was found not competent to stand trial. The government then filed a motion for an order to involuntarily medicate Onuoha with the goal of restoring him to competency, which the district court granted.

Holding: On appeal, the Ninth Circuit vacated and remanded, holding that the district court erred in finding that the proposed treatment was in Onuoha’s best medical interest under the Sell test. The court of appeals held that the district court was correct in finding that there is an important government interest at stake in prosecuting Onuoha, but the district court clearly erred in finding that the proposed course of treatment was in Onuoha’s best medical interests.

Notable Points:

The seriousness of the offense outweighed the “special circumstance” of time detained: The first Sell factor requires the government to prove that important governmental interests are at stake in prosecuting Onuoha. In Sell, the court recognized that there may be some special circumstances that diminish the government’s interest in prosecution, including the amount of time the defendant had already been confined. In this case, the court considered that Onuoha had already spent time in custody since September 2013, amounting to confinement beyond the minimum possible sentence. Nevertheless, the court found it important that a conviction and resulting sentence for the serious crime at issue is significant in terms of general deterrence, not just incapacitation of a specific individual. Here, the court concluded that the government’s valid interest in prosecuting Onuoha outweighed any special circumstances of Onuoha’s detention.

The government did not meet its burden for proving the fourth Sell factor by clear and convincing evidence: The fourth Sell factor requires the government to prove that administration of the drugs is medically appropriate and therefore in Onuoha’s best medical interest in light of his medical condition. The Ninth Circuit held that after hearing the testimony of a medical expert experienced in administering involuntary medication, the district court could not credit the expert’s testimony without exploring contradictory evidence in the record. In this case, contradictory evidence included that the recommended treatment increased the risk of side effects, the dosage proposed was higher than is generally recommended, and the use of the proposed drug does not conform to the community standard of care. Because involuntary medication orders are disfavored in light of the significant liberty interests at stake, and because the record demonstrated that the proposed treatment included dosages higher than generally recommended, the medication was not in Onuoha’s best medical interest.

Found in DMHL Volume 35, Issue 2